1969 AHSME Problems/Problem 35

Revision as of 03:16, 1 October 2014 by Timneh (talk | contribs) (Created page with "== Problem == Let <math>L(m)</math> be the <math>x</math> coordinate of the left end point of the intersection of the graphs of <math>y=x^2-6</math> and <math>y=m</math>, where ...")
(diff) ← Older revision | Latest revision (diff) | Newer revision → (diff)

Problem

Let $L(m)$ be the $x$ coordinate of the left end point of the intersection of the graphs of $y=x^2-6$ and $y=m$, where $-6<m<6$. Let $r=[L(-m)-L(m)]/m$. Then, as $m$ is made arbitrarily close to zero, the value of $r$ is:

$\text{(A) arbitrarily close to } 0\quad\\ \text{(B) arbitrarily close to }\frac{1}{\sqrt{6}}\quad \text{(C) arbitrarily close to }\frac{2}{\sqrt{6}} \quad \text{(D) arbitrarily large} \quad \text{(E) undetermined}$

Solution

$\fbox{B}$

See also

1969 AHSC (ProblemsAnswer KeyResources)
Preceded by
Problem 34
Followed by
Problem 35
1 2 3 4 5 6 7 8 9 10 11 12 13 14 15 16 17 18 19 20 21 22 23 24 25 26 27 28 29 30 31 32 33 34 35
All AHSME Problems and Solutions

The problems on this page are copyrighted by the Mathematical Association of America's American Mathematics Competitions. AMC logo.png